Fundamental Rights & Duties - Questions and Answers

1)   Which of the following fundamental rights are available to friendly aliens?

1) Article 21A
2) Article 20
3) Article 21


a. 1, 2
b. 2, 3
c. 1, 3
d. All of the above
Answer  Explanation 

ANSWER: All of the above

Explanation:

    Fundamental rights for only citizens and not to foreigners are -

    1. Article 15 - Prohibition of discrimination on grounds of religion, race, caste, sex or place of birth.
    2. Article 16 - Equality of opportunity in matters of public employment.
    3. Article 19 - Right to freedom (six rights of speech and expression, assembly, association, movement, residence, and profession).
    4. Article 29 - Protection of language, script and culture of minorities.
    5. Article 30 - Right of minorities to establish and administer educational institutions.

    Fundamental rights for both citizens and friendly aliens are -

    1. Article 14 - Equality before law and equal protection of laws.
    2. Article 17 - Abolition of untouchability and its practice.
    3. Article 18 - Abolition of titles (to foreigners if they hold office of profit under the state).
    4. Article 20 - Protection in respect of conviction for offences.
    5. Article 21 - Protection of life and personal liberty.
    6. Article 21A - Right to elementary education.
    7. Article 22 - Protection against arrest and detention in certain cases.
    8. Article 23 -Prohibition of traffic in human beings and forced labour.
    9. Article 24 - Prohibition of employment of children in factories etc.
    10. Article 25 - Freedom of conscience and free profession, practice and propagation of religion.
    11. Article 26 - Freedom to manage religious affairs.
    12. Article 27 - Freedom from payment of taxes for promotion of any religion.
    13. Article 28 - Freedom from attending religious instruction or worship in certain educational institutions.

  • Enemy Aliens do not enjoy rights under Article 22.


2)   According to article 12 which of the following is the state?

1) CAG
2) Improvement trusts
3) ONGC


a. 1, 2
b. 2, 3
c. 1, 3
d. All of the above
Answer  Explanation 

ANSWER: All of the above

Explanation:

  • Article 12 has defined the term ‘state’ for the purposes of Fundamental Rights i.e., Part III.

  • According to it, the State includes the -

    (a) Government and Parliament of India, that is, executive and legislative organs of the Union government.
    (b) Government and legislature of states, that is, executive and legislative organs of state government.
    (c) All local authorities, that is, municipalities, panchayats, district boards, improvement trusts, etc.
    (d) All other authorities, that is, statutory or non-statutory authorities like LIC, ONGC, SAIL, etc.

  • State has been defined in a wider sense so as to include all its agencies.
  • It is the actions of these agencies that can be challenged in the courts as violating the Fundamental Rights.
  • According to the Supreme Court, even a private body or an agency working as an instrument of the State falls within the meaning of the ‘State’ under Article 12.

  • Article 13 declares that all laws that are inconsistent with or in derogation of any of the fundamental rights shall be void.
  • Thus, it expressively provides for the doctrine of judicial review.

  • The term ‘law’ in Article 13 has been given a wide connotation so as to include -

    (a) Permanent laws enacted by the Parliament or the state legislatures;
    (b) Temporary laws like ordinances issued by the president or the state governors;
    (c) Statutory instruments in the nature of delegated legislation (executive legislation) like order, bye-law, rule, regulation or notification; and
    (d) Non-legislative sources of law, that is, custom or usage having the force of law.

  • Thus, not only a legislation but any of the above can be challenged in the courts as violating a Fundamental Right and hence, can be declared as void.
  • Also, Article 13 says that a constitutional amendment is not a law and hence cannot be challenged.
  • But, the Supreme Court said in the Kesavananda Bharati case (1973) that a Constitutional amendment can be challenged on the ground that it violates a fundamental right that forms a part of the ‘basic structure’ of the Constitution and hence, can be declared as void.


3)   What are the features of the fundamental rights?

1) Law for giving effect to fundamental rights can be made only by the Parliament.
2) Fundamental rights are defended and guaranteed by the Supreme Court.
3) Fundamental rights available are absolute.


a. 2, 3
b. 1, 2
c. 1, 3
d. All of the above
Answer  Explanation 

ANSWER: 1, 2

Explanation:

  • Some of them are available only to the citizens. Others are available to all including foreigners or legal persons like corporations or companies, etc.

  • They are not absolute but qualified. The state can impose reasonable restrictions on them (balance between individual liberty and social control).

  • The courts can decide whether such restrictions are reasonable or not.

  • Most of them are available against the arbitrary action of the State, with a few exceptions like those against the State’s action and against the action of private individuals.

  • When the rights that are available against the State’s action only are violated by the private individuals, there are no constitutional remedies but only ordinary legal remedies.

  • Some of them are negative in character, as they place limitations on the authority of the State,while others are positive as they confer certain privileges on the persons.

  • They are justiciable, allowing persons to move the courts for their enforcement on violation.

  • They are defended and guaranteed by the Supreme Court.

  • They are not permanent and so the Parliament can curtail or repeal them but only by a constitutional amendment act without affecting the ‘basic structure’ of the Constitution.

  • They can be suspended during the operation of a National Emergency except the rights guaranteed by Articles 20 and 21.

  • The six rights guaranteed by Article 19 can be suspended only when emergency is declared on the grounds of war or external aggression(i.e., external emergency) and not on the ground of armed rebellion (i.e., internal emergency).

  • Their scope of operation is limited by Article 31A (saving of laws providing for acquisition of estates, etc.), Article 31B (validation of certain acts and regulations included in the 9thSchedule) and Article 31C (saving of laws giving effect to certain directive principles).

  • Their application to the members of armed forces, para-military forces, police forces,intelligence agencies and analogous services can be restricted or abrogated by the Parliament(Article 33).

  • Their application can be restricted while martial law is in force in any area (Article 34).

  • Most of them are directly enforceable (self-executory) while a few of them can be enforced on the basis of a law made for giving effect to them.

  • Such a law can be made only by the Parliament and not by state legislatures so that uniformity throughout the country is maintained.


4)   Which of the following is/are true regarding Fundamental Rights (FRs)?

1) First written document regarding rights was issued in 1215.
2) Right to property is part of Article 21 that deals with protection of life and personal liberty.


a. Only 1
b. Only 2
c. Both 1 and 2
d. Neither 1 nor 2
Answer  Explanation 

ANSWER: Only 1

Explanation:

  • ‘Magna Carta’ is the Charter of Rights issued by King John of England in 1215 under pressure from the barons.

  • This is the first written document relating to the Fundamental Rights of citizens.

  • The Fundamental Rights are enshrined in Part III of the Constitution from Articles 12 to 35.

  • Constitution of USA (i.e., Bill of Rights) has been inspiration for this.

  • Part III of the Constitution contains a very long and comprehensive list of ‘justiciable’ Fundamental Rights.

  • They are more elaborate than those found in the Constitution of any other country in the world,including the USA.

  • The Fundamental Rights are guaranteed by the Constitution to all persons without any discrimination.

  • They uphold the equality of all individuals, the dignity of the individual, the larger public interest and unity of the nation.

  • The Fundamental Rights are meant for promoting the ideal of political democracy.

  • They prevent the establishment of an authoritarian and despotic rule in the country, and protect the liberties and freedoms of the people against the invasion by the State.

  • The Fundamental Rights are named so because they are guaranteed and protected by the Constitution,which is the fundamental law of the land.

  • They are ‘fundamental’ also in the sense that they are most essential for the all-round development (material, moral, intellectual and spiritual) of the individuals.

  • Originally, the Constitution provided for seven Fundamental Rights viz,

    1. Right to equality (Articles 14-18)
    2. Right to freedom (Articles 19-22)
    3. Right against exploitation (Articles 23-24)
    4. Right to freedom of religion (Articles 25-28)
    5. Cultural and educational rights (Articles 29-30)
    6. Right to property (Article 31)
    7. Right to constitutional remedies (Article 32)

  • But, the right to property was deleted from the list of Fundamental Rights by the 44th Amendment Act, 1978.
  • It is made a legal right under Article 300-A in Part XII of the Constitution.
  • At present, there are only six Fundamental Rights.


5)   Which of the following is exception to rule of equality before law enshrined in Article 14?

1) Chief Minister’s actions
2) Publication in a newspaper of a substantially true report of any proceedings of Parliament
3) Article 31C


a. 1, 3
b. 1, 2
c. 2, 3
d. All of the above
Answer  Explanation 

ANSWER: 2, 3

Explanation:

  • There are constitutional and other exceptions to the rule of equality before law. Thus, it is not absolute.

  • The President of India and the Governor of States enjoy the following immunities (Article 361) -

    1. The President or the Governor is not answerable to any court for the exercise and performance of the powers and duties of his office.

    2. No criminal proceedings shall be instituted or continued against the President or the Governor in any court during his term of office.

    3. No process for the arrest or imprisonment of the President or the Governor shall be issued from any court during his term of office.

    4. No civil proceedings against the President or the Governor shall be instituted during his term of office in any court in respect of any act done by him in his personal capacity,whether before or after he entered upon his office, until the expiration of two months next after notice has been delivered to him.

  • No person shall be liable to any civil or criminal proceedings in any court in respect of the publication in a newspaper (or by radio or television) of a substantially true report of any proceedings of either House of Parliament or either House of the Legislature of a State(Article 361-A).
  • No member of Parliament shall be liable to any proceedings in any court in respect of anything said or any vote given by him in Parliament or any committee thereof (Article 105).

  • No member of the Legislature of a state shall be liable to any proceedings in any court in respect of anything said or any vote given by him in the Legislature or any committee thereof(Article 194).

  • Article 31-C provides that the laws made by the state for implementing the Directive Principles contained in clause (b) or (c) of Article 39 cannot be challenged on the ground that they are in violation of Article 14.

  • The Supreme Court held that “where Article 31-C comes in, Article 14 goes out”.

  • The foreign sovereigns (rulers), ambassadors and diplomats enjoy immunity from criminal and civil proceedings.

  • UNO and its agencies enjoy the diplomatic immunity.


6)   Which of the following cases are related to the interpretation of the Article 21?

1) Gopalan case (1950)
2) Golakhnath case (1967)
3) Menaka case (1978)
4) Kesavananda Bharati case (1973)


a. 1, 3
b. 1, 2
c. 2, 3, 4
d. 1, 3, 4
Answer  Explanation 

ANSWER: 1, 3

Explanation:

  • Article 21 declares that no person shall be deprived of his life or personal liberty except according to procedure established by law.

  • This right is available to both citizens and non-citizens.

  • In Gopalan case (1950), the Supreme Court took a narrow interpretation of the Article 21.

  • It held that the protection under Article 21 is available only against arbitrary executive action and not from arbitrary legislative action.

  • It means that the State can deprive the right to life and personal liberty of a person based on a law.

  • This is because of the expression ‘procedure established by law’ in Article 21, which is different from the expression ‘due process of law’ contained in the American Constitution.

  • Hence, the validity of a law that has prescribed a procedure cannot be questioned on the ground that the law is unreasonable, unfair or unjust.

  • Secondly, the Supreme Court held that the ‘personal liberty’ means only liberty relating to the person or body of the individual.

  • In Menaka case (1978), the Supreme Court overruled its judgement in the Gopalan case by taking a wider interpretation of the Article 21.

  • Therefore, it ruled that the right to life and personal liberty of a person can be deprived by a law provided the procedure prescribed by that law is reasonable, fair and just.

  • In other words, it has introduced the American expression ‘due process of law’.

  • Also, the court held that the ‘right to life’ as embodied in Article 21 is not merely confined to animal existence or survival but it includes within its ambit the right to live with human dignity and all those aspects of life which go to make a man’s life meaningful, complete and worth living.

  • It also ruled that the expression ‘Personal Liberty’ in Article 21 is of the widest amplitude and it covers a variety of rights that go to constitute the personal liberties of a man.

  • SC has declared the following rights as part of Article 21 :

1. Right to live with human dignity.2. Right to decent environment including pollution free water and air and protection against hazardous industries.
3. Right to livelihood.4. Right to privacy.
5. Right to shelter6. Right to health.
7. Right to free education up to 14 years of age.8. Right to free legal aid.
9. Right against solitary confinement.10. Right to speedy trial.
11. Right against handcuffing.12. Right against inhuman treatment.
13. Right against delayed execution.14. Right to travel abroad.
15. Right against bonded labour.16. Right against custodial harassment.
17. Right to emergency medical aid.18. Right to timely medical treatment in government hospital.
19. Right not to be driven out of a state.20. Right to fair trial.
21. Right of prisoner to have necessities of life.22. Right of women to be treated with decency and dignity.
23. Right against public hanging.24. Right to hearing.
25. Right to information.26. Right to reputation.



7)   Which of the following is/are true regarding right to education?

1) Article 21 deals with right to education.
2) 86th Constitutional Amendment Act, 2002 changed article 45.


a. Only 1
b. Only 2
c. Both 1 and 2
d. Neither 1 nor 2
Answer  Explanation 

ANSWER: Only 2

Explanation:

  • Article 21 A declares that the State shall provide free and compulsory education to all children of the age of six to fourteen years in such a manner as the State may determine.

  • This provision makes only elementary education a Fundamental Right and not higher or professional education.

  • This was added by the 86th Constitutional Amendment Act of 2002.

  • Even before this, the Constitution had a provision for free and compulsory education for children under Article 45 in Part IV.

  • But being a directive principle, it was not enforceable by the courts.

  • Now, there is scope for judicial intervention in this regard.

  • It also changed subject matter of Article 45 which now reads - ‘The state shall endeavour to provide early childhood care and education for all children until they complete the age of six years.’

  • It also added a new fundamental duty under Article 51A that reads - ‘It shall be the duty of every citizen of India to provide opportunities for education to his child or ward between the age of six and fourteen years’.

  • In 1993 itself, the Supreme Court recognized a Fundamental Right to primary education in the right to life under Article 21.

  • It held that every child or citizen of this country has a right to free education until he completes the age of 14 years.

  • After that, his right to education is subject to the limits of economic capacity and development of the state.

  • Thus, the Court overruled its earlier judgement (1992) which declared that there was a fundamental right to education up to any level including professional education like medicine and engineering.

  • In pursuance of Article 21A, the Parliament enacted the Right of Children to Free and Compulsory Education (RTE) Act, 2009.

  • This Act says that every child has a right to be provided full time elementary education of satisfactory and equitable quality in a formal school which satisfies certain essential norms and standards.

  • The 86th Constitutional Amendment Act, 2002 and the Right of Children to Free and Compulsory Education Act, 2009 have come into force w.e.f. 1 April 2010.


8)   Which is true?

a. Protection against self-incrimination extends to compulsion to give thumb impression.
b. The protection against double jeopardy is available in proceedings before a court of law or administrative authorities.
c. Protection under Article 20(first provision) cannot be claimed in case of preventive detention.
d. Criminal and Civil law cannot punish retrospectively.
Answer  Explanation 

ANSWER: Protection under Article 20(first provision) cannot be claimed in case of preventive detention.

Explanation:

  • Article 20 grants protection against arbitrary and excessive punishment to an accused person, whether citizen or foreigner or legal person like a company or a corporation.

  • It contains 3 provisions -
    (a) No ex-post-facto law : No person shall be
    (i) convicted of any offence except for violation of a law in force at the time of the commission of the act; nor
    (ii) subjected to a penalty greater than that prescribed by the law in force at the time of the commission of the act.

    (b) No double jeopardy : No person shall be prosecuted and punished for the same offence more than once.

    (c) No self-incrimination : No person accused of any offence shall be compelled to be a witness against himself.

  • An ex-post-facto law is one that imposes penalties retrospectively (retroactively), that is, upon acts already done or which increases the penalties for such acts

  • The enactment of such a law is prohibited by the first provision of Article 20.

  • But this is only for criminal laws.

  • A civil law/liability or a tax can be imposed retrospectively.

  • Also, this provision prohibits only conviction or sentence under an ex-post-facto criminal law and not the trial thereof.

  • Finally, the protection (immunity) under this provision cannot be claimed in case of preventive detention or demanding security from a person.

  • The protection against double jeopardy is available only in proceedings before a court of law or a judicial tribunal.

  • It is not available in proceedings before departmental or administrative authorities as they are not judicial in nature.

  • The protection against self-incrimination extends to both oral evidence and documentary evidence.

  • But, it does not extend to -
  • (i) compulsory production of material objects;
    (ii) compulsion to give thumb impression, specimen signature, blood specimens; and
    (iii) compulsory exhibition of the body.

  • It extends only to criminal proceedings and not to civil proceedings or proceedings that are not criminal in nature.


9)   Which of the following is/are true?

1) Article 17 deals with abolition of titles.
2) Bharat Ratna, Padma Vibhushan, Padma Bhushan and Padma Sri cannot be used as suffixes or prefixes to the names of awardees.


a. Only 1
b. Only 2
c. Both 1 and 2
d. Neither 1 nor 2
Answer  Explanation 

ANSWER: Only 2

Explanation:

  • Article 18 abolishes titles.

    It also makes 4 provisions -

  • (a) It prohibits the state from conferring any title (except a military or academic distinction) on anybody, whether a citizen or a foreigner.

    (b) It prohibits a citizen of India from accepting any title from any foreign state.

    (c) A foreigner holding any office of profit or trust under the state cannot accept any title from any foreign state without the consent of the president.

    (d) No citizen or foreigner holding any office of profit or trust under the State is to accept any present, emolument or office from or under any foreign State without the consent of the president.

  • Hereditary titles of nobility like Maharaja, Rai Bahadur, Rai Saheb, Dewan Bahadur etc.. which were conferred by colonial States are banned as these are against the principle of equal status of all.

  • In 1996, the Supreme Court upheld the constitutional validity of the National Awards-Bharat Ratna, Padma Vibhushan, Padma Bhushan and Padma Sri.

  • It said that these awards do not amount to ‘titles’ within the meaning of Article 18 that prohibits only hereditary titles of nobility.

  • They do not violate Article 18 as the theory of equality does not mandate that merit should not be recognized.

  • But, they should not be used as suffixes or prefixes to the names of awardees. Otherwise, the awardees should forfeit the awards.

  • These National Awards were instituted in 1954.

  • The Janata Party government headed by Morarji Desai discontinued them in 1977. But they were again revived in 1980 by the Indira Gandhi government.


10)   Which of the following is/are true?

1) Refusing to sell goods or render services to any person is not considered an offence under ambit of Article 17.
2) The term ‘untouchability’ has been defined in the Constitution in Article 17 dealing with abolition of untouchability.


a. Only 1
b. Only 2
c. Both 1 and 2
d. Neither 1 nor 2
Answer  Explanation 

ANSWER: Neither 1 nor 2

Explanation:

  • Article 17 abolishes ‘untouchability’ and forbids its practice in any form.

  • The enforcement of any disability arising out of untouchability shall be an offence punishable in accordance with law.

  • In 1976, the Untouchability (Offences) Act, 1955 was amended and renamed as the Protection of Civil Rights Act, 1955 to enlarge the scope and make penal provisions more stringent.

  • The act defines civil right as any right accruing to a person by reason of the abolition of untouchability by Article 17 of the Constitution.

  • The term ‘untouchability’ has not been defined either in the Constitution or in the Act.

  • Mysore High Court held that the subject matter of Article 17 is not untouchability in its literal but the ‘practice as it had developed historically in the country’.

  • It refers to the social disabilities imposed on certain classes of persons by reason of their birth in certain castes.

  • Hence, it does not cover social boycott of a few individuals or their exclusion from religious services, etc.

  • Under the Protection of Civil Rights Act (1955), the offences committed on the ground of untouchability are punishable either by imprisonment up to six months or by fine up to `500 or both.

  • A person convicted of the offence of ‘untouchability’ is disqualified for election to the Parliament or state legislature.

  • The act declares the following acts as offences :

    (a) preventing any person from entering any place of public worship or from worshipping therein;

    (b) justifying untouchability on traditional, religious, philosophical or other grounds;

    (c) denying access to any shop, hotel or places of public entertainment;

    (d) insulting a person belonging to scheduled caste on the ground of untouchability;

    (e) refusing to admit persons in hospitals, educational institutions or hostels established for public benefit;

    (f) preaching untouchability directly or indirectly; and

    (g) refusing to sell goods or render services to any person.

  • The Supreme Court held that the right under Article 17 is available against private individuals.

  • It is the constitutional obligation of the State to take necessary action to ensure that this right is not violated.


11)   Which of the following are criticisms of the Fundamental Rights (FRs)?

1) They are against authority
2) Parliament can curtail or abolish them
3) Feature of preventive detention
4) Complex language


a. Only 1, 2 and 4
b. Only 2 and 3
c. Only 2, 3 and 4
d. All of the above
Answer  Explanation 

ANSWER: Only 2, 3 and 4

Explanation:

    The criticisms are -

    (a) Excessive Limitations - They are subjected to innumerable exceptions, restrictions, qualified and not absolute, etc. Thus,the critics remarked that the Constitution grants Fundamental Rights with one hand and takes them away with the other.

    (b) No Social and Economic Rights in the list like right to social security, right to work, right to employment, right to rest and leisure, etc. which are made available to the citizens of advanced democratic countries. Also, the socialistic constitutions of erstwhile USSR or China provided for such rights.

    (c) No Clarity -They are stated in a vague, unclear and ambiguous manner.

    (d) The language used is very complicated and beyond the comprehension of the common man. It is alleged that the Constitution was made by the lawyers for the lawyers.

    (e) No Permanency - They are not sacrosanct as Parliament can curtail or abolish them. For example,the abolition of the fundamental right to property in 1978.

    (f) Thus, they can become a tool in the hands of politicians having majority support in the Parliament.

    (g) Suspension During Emergency - The suspension of their enforcement during the operation of National Emergency (except Articles 20 and 21) cuts at the roots of democratic system in India by placing the rights of the innocent in continuous jeopardy.

    (h) According to the critics, the Fundamental Rights should be enjoyable in all situations - Emergency or no Emergency.

    (i) Expensive Remedy - The judiciary has been made responsible for defending and protecting these rights.

    (j) However, the judicial process is too expensive and time consuming benefitting the rich.

    (k) Preventive Detention (Article 22) takes away the spirit of the fundamental rights conferring arbitrary powers on the State and negates individual liberty.

    (l) It justifies the criticism that the Constitution of India deals more with the rights of the State against the individual than with the rights of the individual against the State.

    (m) Notably, no democratic country in the world has made preventive detention as an integral part of their Constitutions as has been made in India.

    (n) No Consistent Philosophy - According to some critics, FRs are not product of any philosophical principle, which creates difficulty for the Supreme Court and the high courts in interpreting the fundamental rights.

    The significance are -

    (a) They form bedrock of democratic system in India
    (b) Provide necessary conditions for the material and moral protection of man.
    (c) Serve as a formidable bulwark of individual liberty.
    (d) Facilitate the establishment of rule of law
    (e) Protect the interests of minorities and weaker sections of society.
    (f) They strengthen the secular fabric of India.
    (g) Check absoluteness of the authority of government.
    (h) Lay down the foundation stone of social equality and social justice.
    (i) They ensure the dignity and respect of individuals.
    (j) Facilitate participation of people in the political and administrative process.


12)   Which of the following are legal rights?

1) Right to property
2) No tax shall be levied or collected except by authority of law
3) Freedom of Trade, commerce and intercourse


a. Only 1 and 2
b. Only 1 and 3
c. Only 2 and 3
d. All of the above
Answer  Explanation 

ANSWER: All of the above

Explanation:

  • Besides the Fundamental Rights, there are certain other rights contained in other parts of the Constitution.

  • These rights are known as constitutional rights or legal rights or non-fundamental rights.

  • They are :

    1. No tax shall be levied or collected except by authority of law (Article 265 in Part XII).

    2. No person shall be deprived of his property save by authority of law (Article 300-A in Part XII).

    3. Trade, commerce and intercourse throughout the territory of India shall be free (Article 301 in Part XIII).

    4. The elections to the Lok Sabha and the State Legislative Assembly shall be on the basis of adult suffrage (Article 326 in Part XV).

  • These rights are also equally justiciable.

  • In case of violation of a Fundamental Right, the aggrieved person can directly move the Supreme Court for its enforcement under Article 32, which is in itself a fundamental right.

  • However, in case of violation of the above rights, the aggrieved person cannot avail this constitutional remedy but can move the High Court by an ordinary suit or under Article 226 (writ jurisdiction of high court).


13)   Which of the following is/are true?

1) No citizen can be discriminated against or be ineligible for any employment under the State on grounds of residence.
2) Parliament can prescribe residence as a condition for appointment in a state.


a. Only 1
b. Only 2
c. Both 1 and 2
d. Neither 1 nor 2
Answer  Explanation 

ANSWER: Both 1 and 2

Explanation:

  • Article 16 provides for equality of opportunity for all citizens in matters of employment or appointment to any office under the State.

  • No citizen can be discriminated against or be ineligible for any employment or office under the State on grounds of only religion, race, caste, sex, descent, place of birth or residence.

  • There are 2 exceptions to this -

    1. Parliament can prescribe residence as a condition for certain employment or appointment in a state or union territory or local authority or other authority.

    2. As the Public Employment (Requirement as to Residence) Act of 1957 expired in 1974, there is no such provision for any state except Andhra Pradesh. (By article 371D added by 32nd Amendment Act 1973)

    3. The State can provide for reservation of appointments or posts in favor of any backward class that is not adequately represented in the state services.

    4. A law can provide that the incumbent of an office related to religious or denominational institution or a member of its governing body should belong to the particular religion or denomination.


14)   Equality before law is part of Rule of Law. Rule of Law was propounded by -

a. Napolean
b. Voltaire
c. A. V. Dicey
d. Ivor Jennings
Answer  Explanation 

ANSWER: A. V. Dicey

Explanation:

  • Article 14 says that the State shall not deny to any person equality before the law or the equal protection of the laws within the territory of India.

  • This right is for all whether citizens or foreigners.

  • Moreover, the word ‘person’ includes legal persons, viz, statutory corporations, companies, registered societies or any other type of legal person.

  • The concept of ‘equality before law’ is of British origin.

  • But the concept of ‘equal protection of laws’ has been taken from the American Constitution.

  • The first concept connotes -

    1. the absence of any special privileges in favor of any person;
    2. the equal subjection of all persons to the ordinary law of the land administered by ordinary law courts; and
    3. no person is above the law.

    The second concept, on the other hand, connotes -

    1. the equality of treatment under equal circumstances, both in the privileges conferred and liabilities imposed by the laws,
    2. the similar application of the same laws to all persons who are similarly situated, and
    3. the like should be treated alike without any discrimination.

  • Thus, the former is a negative concept while the latter is a positive concept. However, both of them aim at establishing equality of legal status, opportunity and justice.

  • The Supreme Court held that where equals and unequals are treated differently, Article 14 does not apply.

  • While Article 14 forbids class legislation, it permits reasonable classification of persons,objects and transactions by the law. But the classification should not be arbitrary, artificial or evasive.

  • Rather, it should be based on an intelligible differential and substantial distinction.

  • Rule of Law The concept of ‘equality before law’ is an element of the concept ‘Rule of Law’, propounded by A.V. Dicey, the British jurist.

  • His concept has following 3 aspects -

    1. Absence of arbitrary power - no man can be punished except for a breach of law.

    2. Equality before the law - equal subjection of all citizens (rich or poor, high or low,official or non-official) to the ordinary law of the land administered by the ordinary law courts.

    3. The primacy of the rights of the individual - the constitution is the result of the rights of the individual as defined and enforced by the courts of law rather than the constitution being the source of the individual rights.

  • The first and the second elements are applicable to India and not the third.

  • In India, the constitution is the source of the individual rights.

  • The Supreme Court held that the ‘Rule of Law’ as embodied in Article 14 is a ‘basic feature’ of the constitution. Hence, it cannot be destroyed even by an amendment.


15)   Article 30 grants rights to

1) Religious minorities
2) Linguistic minorities
3) Cultural Minorities


a. Only 1 and 2
b. Only 1 and 3
c. Only 2 and 3
d. All of the above
Answer  Explanation 

ANSWER: Only 1 and 2

Explanation:

    Article 30 grants the following rights to minorities, whether religious or linguistic :

    1. All minorities shall have the right to establish and administer educational institutions of their choice.

    2. The compensation amount fixed by the State for the compulsory acquisition of any property of a minority educational institution shall not restrict or abrogate the right guaranteed to them.

    3. This above provision was added by the 44thAmendment Act of 1978 to protect the right of minorities in this regard.

    4. The Act deleted the right to property as a Fundamental Right (Article31).

    5. In granting aid, the State shall not discriminate against any educational institution managed by a minority.

  • Thus, the protection under Article 30 is confined only to minorities (religious or linguistic) and does not extend to any section of citizens (as under Article 29).

  • However, the term ‘minority’ has not been defined anywhere in the Constitution.

  • The right under Article 30 also includes the right of a minority to impart education to its children in its own language.

  • Minority educational institutions are of three types :

    1. institutions that seek recognition as well as aid from the State;
    2. institutions that seek only recognition from the State and not aid; and
    3. institutions that neither seek recognition nor aid from the State.

  • The institutions of 1st and 2nd type are subject to the regulatory power of the state with regard to syllabus prescription, academic standards, discipline, sanitation, employment of teaching staff and soon.

  • The institutions of 3rdtype are free to administer their affairs but subject to operation of general laws like contract law, labour law, industrial law, tax law, economic regulations, and so on.


16)   Which of the following is/are true regarding freedom of religion?

1) The word Hindus in Article 25 does not include Sikhs, Jains and Buddhists.
2) Article 26 rights are subject to other provisions relating to the Fundamental Rights.


a. Only 1
b. Only 2
c. Both 1 and 2
d. Neither 1 nor 2
Answer  Explanation 

ANSWER: Neither 1 nor 2

Explanation:
Freedom of Conscience and Free Profession, Practice and Propagation of Religion -

  • Article 25 says that all persons are equally entitled to freedom of conscience and the right to freely profess, practice and propagate religion.

  • There is -

    1. Freedom of conscience : Inner freedom of an individual to mold his relation with God, etc. in whatever way he desires.

    2. Right to profess : Declare one’s religious beliefs and faith openly and freely.

    3. Right to practice : Perform religious worship, rituals, ceremonies and exhibition of beliefs and ideas.

    4. Right to propagate : Transmission and dissemination of one’s religious beliefs to others or exposition of the tenets of one’s religion.

    5. However, it does not include a right to convert another person to one’s own religion.

    6. Forcible conversions impinge on the ‘freedom of conscience’ guaranteed to all the persons alike.

  • Article 25 covers religious beliefs (doctrines) as well as religious practices (rituals).

  • These rights are available to all - citizens as well as non-citizens.

  • But, these rights are subject to public order, morality, health and other provisions relating to fundamental rights.

  • Also, the State can -

    1. regulate or restrict any economic, financial, political or other secular activity associated with religious practice; and

    2. provide for social welfare and reform or throw open Hindu religious institutions of a public character to all classes and sections of Hindus.

    Article 25 also contains two explanations -

    1. wearing and carrying of kirpans is to be included in the profession of the Sikh religion; and

    2. the Hindus, in this context, include Sikhs, Jains and Buddhists.
  • Freedom to Manage Religious Affairs.

  • According to Article 26, every religious denomination or any of its section shall have the following -

    Rights :

    1. Right to establish and maintain institutions for religious and charitable purposes;

    2. Right to manage its own affairs in matters of religion;

    3. Right to own and acquire movable and immovable property; and

    4. Right to administer such property in accordance with law.

  • Article 25 guarantees rights of individuals, while Article 26 guarantees rights of religious denominations or their sections.

  • Article 26 protects collective freedom of religion.

  • Article 26 rights are also subject to public order, morality and health but not subject to other provisions relating to the Fundamental Rights.

  • The Supreme Court held that a religious denomination must satisfy 3 conditions -

    1. It should be a collection of individuals who have a system of beliefs (doctrines) which they regard as conductive to their spiritual well-being;

    2. It should have a common organization; and

    3. It should be designated by a distinctive name.

  • Supreme Court said that the ‘Ramakrishna Mission’ and ‘Ananda Marga’ are religious denominations within the Hindu religion.

  • It also said that Aurobindo Society is not a religious denomination.


17)   Which of the following is/are true?

1) In institutions recognized by the State religious instruction is completely prohibited
2) Article 27 prohibits levy of tax and fees.
3) Taxes can be used for the promotion or maintenance of all religions.


a. 2, 3
b. 1, 2
c. 1, 3
d. All of the above
Answer  Explanation 

ANSWER: 2, 3

Explanation:
Freedom from Taxation for Promotion of a Religion

  • Article 27 says that no person shall be compelled to pay any taxes for the promotion or maintenance of any particular religion or religious denomination.

  • State should not spend the public money collected by way of tax for the promotion or maintenance of any particular religion.

  • This prohibits the State from favoring and supporting any one religion over the other.

  • Taxes can be used for the promotion or maintenance of all religions.

  • This provision prohibits only levy of a tax and not a fee.

  • This is because the purpose of a fee is to control secular administration of religious institutions and not to promote or maintain religion.

  • Thus, a fee can be levied on pilgrims to provide them some special service or safety measures.

  • Similarly, a fee can be levied on religious endowments for meeting the regulation expenditure.

  • Freedom from Attending Religious Instruction -

  • Under Article 28, no religious instruction shall be provided in any educational institution wholly maintained out of State funds.

  • But, this provision shall not apply to an educational institution administered by the State but established under any endowment or trust, requiring imparting of religious instruction in such institution.

  • No person attending any educational institution recognized by the State or receiving aid out of State funds shall be required to attend any religious instruction or worship in that institution without his consent.

  • In case of a minor, the consent of his guardian is needed.

  • Article 28 distinguishes between four types of educational institutions -

    1. Institutions wholly maintained by the State.

    2. Institutions administered by the State but established under any endowment or trust.

    3. Institutions recognized by the State.

    4. Institutions receiving aid from the State.

  • In (a) religious instruction is completely prohibited while in (b), religious instruction is permitted.

  • In (c) and (d), religious instruction is permitted on a voluntary basis.


18)   Which of the following is/are true?

1) ‘Begar’ means compelling a person to work against his will.
2) Article 24 deals with trafficking in human beings.


a. Only 1
b. Only 2
c. Both 1 and 2
d. Neither 1 nor 2
Answer  Explanation 

ANSWER: Neither 1 nor 2

Explanation:
Prohibition of Traffic in Human Beings and Forced Labour -

  • Article 23 prohibits traffic in human beings, begar (forced labor) and other similar forms of forced labor.

  • This right is available to both citizens and non-citizens.

  • It protects the individual not only against theState but also against private persons.

  • The expression ‘traffic in human beings’ include -

    1. selling and buying of men, women and children like goods;

    2. immoral traffic in women and children, including prostitution;

    3. deva-dasis; and

    4. slavery

  • To punish, the Parliament has made the Immoral Traffic (Prevention) Act, 1956.

  • The term ‘begar’ means compulsory work without remuneration.

  • It was a peculiar Indian system under which the local zamindars sometimes used to force their tenants to render services without payment.

  • In addition to begar, the Article 23 prohibits other ‘similar forms of forced labor’ like ‘bonded labor’.

  • The term ‘forced labor’ means compelling a person to work against his will.

  • The word ‘force’ includes not only physical or legal force but also force arising from the compulsion of economic circumstances, i.e. working for less than the minimum wage.

  • The Bonded Labour System (Abolition) Act, 1976; the Minimum Wages Act, 1948; the Contract Labour Act, 1970and the Equal Remuneration Act, 1976 were made.

  • Article 23 also provides for an exception i.e., it permits the State to impose compulsory service for public purposes, as for e.g. military service or social service, for which it is not bound to pay.

  • However, in imposing such service, the State is not permitted to make any discrimination on grounds only of religion, race, caste or class.


19)   Which of the following is/are true?

1) Children can work in family business but after school hours.
2) Employing children below 14 years in hotels is banned
3) Presently, children under the age of 14 are prohibited from employment in hazardous occupations and processes while employment in non-hazardous occupations is regulated.


a. 1, 3
b. 1, 2
c. 2, 3
d. All of the above
Answer  Explanation 

ANSWER: 1, 2

Explanation:
Prohibition of Employment of Children in Factories etc. -

  • Article 24 prohibits the employment of children below the age of 14 years in any factory, mine or other hazardous activities like construction work or railway.

  • But it does not prohibit their employment in any harmless or innocent work.

  • The Child Labour (Prohibition and Regulation) Act, 1986, is the most important law for this.

  • The Employment of Children Act, 1938; the Factories Act, 1948; the Mines Act,1952; the Merchant Shipping Act, 1958; the Plantation Labour Act, 1951; the Motor Transport Workers Act, 1951; Apprentices Act, 1961; the Bidi and Cigar Workers Act, 1966; and other similar acts prohibit the employment of children below certain age.

  • In 1996, the Supreme Court directed the establishment of Child Labour Rehabilitation Welfare Fund in which the offending employer should deposit a fine of Rs. 20,000 for each child employed by him.

  • It also issued directions for the improvement of education, health and nutrition of children.

  • The Commissions for Protection of Child Rights Act, 2005 was enacted to provide for the establishment of a National Commission and State Commissions for Protection of Child Rights.

  • Children’s Courts for providing speedy trial of offences against children or of violation of child rights are provided.

  • In 2006, the government banned the employment of children as domestic servants or workers in business establishments like hotels, dhabas, restaurants, shops, factories, resorts, spas, tea-shops and so on.

  • It warned that anyone employing children below 14 years of age would be liable for prosecution and penal action.

  • Complete Ban on Child Labour

    1. In August 2012, the Union Cabinet approved a proposal to completely ban employment of children below 14 years in all occupations and processes.

    2. The Child Labour (Prohibition & Regulation) Act, 1986, will be amended to incorporate the changes and will be renamed a Child and Adolescent Labour (Prohibition) Act.

    3. Presently, children under the age of 14 are prohibited from employment in “hazardous occupations and processes” while their conditions of work in non-hazardous occupations and processes are merely regulated.

    4. There has been increase in the age of prohibition for employment of children and adolescents in hazardous occupations, such as mining, from 14 to 18.

    5. Employment of children below 14 years is presently prohibited in 18 occupations and 65 processes.

    6. The maximum punishment for offences under the Act has been increased from one year to two years of imprisonment and from `20,00 to `50,000 fine or both.

    7. For repeated offences, it has been raised to three years of imprisonment.

    8. Children under 14 years can work in non-hazardous “family enterprises” after school hours and during vacations.

    9. The family is defined to include not just the child’s parents and siblings, but also siblings of the child’s parents.

    10. And a family enterprise includes any work, profession or business in which any family member works along with other persons.


20)   Which of the following articles extends the competence of Parliament to make a law on matters, even though some of those matters may fall within the sphere of State List?

a. 34
b. 35
c. 31
d. 33
Answer  Explanation 

ANSWER: 35

Explanation:

  • Article 35 says that the power to make laws, to give effect to certain specified fundamental rights shall vest only in the Parliament and not in the state legislatures.

  • This provision ensures that there is uniformity throughout India with regard to the nature of those fundamental rights and punishment for their infringement.

  • Article 35 contains the following provisions -

    A. The Parliament shall have (and the legislature of a state shall not have) power to make laws with respect to the following matters :

    1. Prescribing residence as a condition for certain employments or appointments in a state or union territory or local authority or other authority (Article 16).

    2. Empowering courts other than the Supreme Court and the high courts to issue directions,orders and writs of all kinds for the enforcement of fundamental rights (Article 32).

    3. Restricting or abrogating the application of Fundamental Rights to members of armed forces,police forces, etc. (Article 33).

    4. Indemnifying any government servant or any other person for any act done during the operation of martial law in any area (Article 34).

    B. Parliament shall have (and the legislature of a state shall not have) powers to make laws for prescribing punishment for those acts that are declared to be offences under the fundamental rights.

    These include the following :

    1. Untouchability (Article 17).

    2. Traffic in human beings and forced labor (Article 23).

    C. The Parliament shall, after the commencement of the Constitution, make laws for prescribing punishment for the above acts, thus making it obligatory for the Parliament to enact such laws.

    D. Any law in force at the commencement of the Constitution with respect to any of the matters specified above is to continue in force until altered or repealed or amended by the Parliament.

    E. It should be noted that Article 35 extends the competence of the Parliament to make a law on the matters specified above, even though some of those matters may fall within the sphere of the state legislatures (i.e., State List).


21)   Right to property was originally dealt in -

1) Article 33
2) Article 31
3) Article 19
4) Article 21


a. Only 2
b. Only 2 and 3
c. Only 1, 2, and 3
d. All of the above
Answer  Explanation 

ANSWER: Only 2 and 3

Explanation:

  • Originally, the right to property was one of the seven fundamental rights under Part III of the Constitution.

  • It was dealt by Article 19(1)(f) and Article 31.

  • Article 19(1)(f) guaranteed to every citizen the right to acquire, hold and dispose of property.

  • Article 31, on the other hand, guaranteed to every person, whether citizen or non-citizen, right against deprivation of his property.

  • It provided that no person shall be deprived of his property except by authority of law.

  • It empowered the State to acquire or requisition the property of a person on two conditions :

    1. it should be for public purpose; and

    2. it should provide for payment of compensation (amount) to the owner.

  • Since the commencement of the Constitution, the Fundamental Right to Property has been the most controversial.

  • It has caused confrontations between the Supreme Court and the Parliament.

  • Most of the litigation centered around the obligation of the state to pay compensation for acquisition or requisition of private property.

  • Therefore, the 44th Amendment Act of 1978 abolished the right to property as a Fundamental Right by repealing Article 19(1)(f) and Article 31 from Part III.

  • Instead, the Act inserted a new Article 300A in Part XII under the heading ‘Right to Property’.

  • It provides that no person shall be deprived of his property except by authority of law.

  • Thus, the right to property still remains a legal right or a constitutional right, though no longer a fundamental right.

  • It is not a part of the basic structure of the Constitution.

  • The right to property as a legal right (as distinct from the Fundamental Rights) has the following implications :

    1. It can be regulated i.e., curtailed, abridged or modified without constitutional amendment by an ordinary law of the Parliament.

    2. It protects private property against executive action but not against legislative action.

    3. In case of violation, the aggrieved person cannot directly move the Supreme Court under Article 32 (right to constitutional remedies including writs) for its enforcement. He can move the High Court under Article 226.

    4. No guaranteed right to compensation in case of acquisition or requisition of the private property by the state.

  • Though the Fundamental Right to Property under Part III has been abolished, the Part III still carries two provisions which provide for the guaranteed right to compensation in case of acquisition or requisition of the private property by the state.

  • These two cases where compensation has to be paid are :

    1. When the State acquires the property of a minority educational institution (Article 30); and

    2. When the State acquires the land held by a person under his personal cultivation and the land is within the statutory ceiling limits (Article 31 A).

  • The first provision was added by the 44th Amendment Act (1978), while the second provision was added by the 17th Amendment Act (1964).

  • Further, Articles 31A, 31B and 31C have been retained as exceptions to the fundamental rights.


22)   Which article confers the right to remedies for the enforcement of the fundamental rights?

a. 32
b. 31
c. 36
d. 35
Answer  Explanation 

ANSWER: 32

Explanation:

  • Article 32 confers the right to remedies for the enforcement of the fundamental rights of an aggrieved citizen.

  • The right to get the Fundamental Rights protected is in itself a fundamental right.

  • This makes the fundamental rights real.

  • That is why Dr. Ambedkar called Article 32 as the most important article of the Constitution - ‘an Article without which this constitution would be a nullity. It is the very soul of the Constitution and the very heart of it’.

  • The Supreme Court has ruled that Article 32 is a basic feature of the Constitution.

  • Hence, it cannot be abridged or taken away even by way of an amendment to the Constitution.

  • It contains the following four provisions :

    1. The right to move the Supreme Court by appropriate proceedings for the enforcement of the Fundamental Rights is guaranteed.

    2. The Supreme Court shall have power to issue directions or orders or writs for the enforcement of any of the fundamental rights. The writs issued may include habeas corpus, mandamus, prohibition, certiorari and quo-warranto.

    3. Parliament can empower any other court to issue directions, orders and writs of all kinds. However, this can be done without prejudice to the above powers conferred on the Supreme Court. Any other court here does not include high courts because Article 226 has already conferred these powers on the high courts.

    4. The right to move the Supreme Court shall not be suspended except as otherwise provided for by the Constitution. Thus, the Constitution provides that the President can suspend the right to move any court for the enforcement of the fundamental rights during a national emergency (Article 359).

  • It is thus clear that the Supreme Court has been constituted as the defender and guarantor of the fundamental rights of the citizens.

  • It has been vested with the ‘original’ and ‘wide’ powers for that purpose.

  • Original, because an aggrieved citizen can directly go to the Supreme Court, not necessarily by way of appeal.

  • Wide, because its power is not restricted to issuing of orders or directions but also writs of all kinds.

  • The purpose of Article 32 is to provide a guaranteed, effective, expedious, inexpensive and summary remedy for the protection of the fundamental rights.

  • Only the Fundamental Rights guaranteed by the Constitution can be enforced under Article 32 and not any other right like non-fundamental constitutional rights, statutory rights, customary rights and so on.

  • The violation of a fundamental right is the sine qua non for the exercise of the right conferred by Article 32.
  • In other words, the Supreme Court, under Article 32, cannot determine a question that does not involve Fundamental Rights.

  • Article 32 cannot be invoked simply to determine the constitutionality of an executive order or a legislation unless it directly infringes any of the fundamental rights. In case of the enforcement of Fundamental Rights, the jurisdiction of the Supreme Court is original but not exclusive.

  • It is concurrent with the jurisdiction of the high court under Article 226.

  • It vests original powers in the high court to issue directions, orders and writs of all kinds for the enforcement of the Fundamental Rights.

  • It means when the Fundamental Rights of a citizen are violated, the aggrieved party has the option of moving either the high court or the Supreme Court directly.

  • Since the right guaranteed by Article 32 (i.e., the right to move the Supreme Court where a fundamental right is infringed) is in itself a fundamental right, the availability of alternate remedy is no bar to relief under Article 32.

  • However, the Supreme Court has ruled that where relief through high court is available under Article 226, the aggrieved party should first move the high court.


23)   Which of the following are considered exceptions to Fundamental Rights?

1) 31A
2) 31B
3) 31C


a. 1, 3
b. 1, 2
c. 2, 3
d. All of the above
Answer  Explanation 

ANSWER: All of the above

Explanation:

  • Article 31A - Saving of Laws Providing for Acquisition of Estates, etc. (Added by the 1st Constitutional Amendment Act of 1951 and amended by 4th, 17th and 44th Amendments).

  • Article 31A saves five categories of laws from being challenged and invalidated on the ground of contravention of the fundamental rights conferred by Article 14 and Article 19.

  • They are related to agricultural land reforms, industry and commerce and include the following -

    1. Acquisition of estates and related rights by the State;
    2. Taking over the management of properties by the State;
    3. Amalgamation of corporations;
    4. Extinguishment or modification of rights of directors or shareholders of corporations; and
    5. Extinguishment or modification of mining leases.

  • Article 31A does not immunize a state law from judicial review unless it has been reserved for the president’s consideration and has received his assent.

  • This Article also provides for the payment of compensation at market value when the state acquires the land held by a person under his personal cultivation and the land is within the statutory ceiling limit.

  • Article 31B - Validation of Certain Acts and Regulations (Article 31B and 9th Schedule added by 1st Constitutional Amendment Act of 1951).

  • Article 31B saves the acts and regulations included in the 9th Schedule from being challenged and invalidated on the ground of contravention of any of the fundamental rights.

  • Scope of Article 31B is wider than Article 31A.

  • Article 31B immunizes any law included in the 9th Schedule from all the fundamental rights and not just 5 categories in Article 31A.

  • However, in January 2007, the Supreme Court ruled that there could not be any blanket immunity from judicial review of laws included in the 9th Schedule.

  • The court held that judicial review is a ‘basic feature’ of the constitution and laws placed under the Ninth Schedule after April 24, 1973, are open to challenge in court if they violated fundamentals rights guaranteed under Articles 14, 15, 19 and 21 or the ‘basic structure’ of the constitution.

  • It was on April 24, 1973, that the Supreme Court first propounded the doctrine of ‘basic structure’ or ‘basic features’ of the constitution in its landmark verdict in the Kesavananda Bharati Case.

  • Originally (in 1951), the 9th Schedule contained only 13 acts and regulations but at present (in 2013) their number is 282.

  • Article 31C - Saving of Laws Giving Effect to Certain Directive Principles

  • Article 31C, as inserted by the 25th Amendment Act of 1971, contained the following two provisions :

    (a) No law that seeks to implement the socialistic directive principles specified in Article 39(b) or (c) shall be void on the ground of contravention of the fundamental rights conferred by Article 14 or Article 19.

    (b) No law containing a declaration that it is for giving effect to such policy shall be questioned in any court on the ground that it does not give effect to such a policy.

    In the Kesavananda Bharati case (1973), the Supreme Court declared the above 2nd provision of Article 31C as unconstitutional and invalid on the ground that judicial review is a basic feature of the Constitution and hence, cannot be taken away.

    However, the above 1st provision of Article 31C was held to be constitutional and valid.

    The 42nd Amendment Act (1976) extended the scope of the above 1stprovision of Article 31C by including within its protection any law to implement any of the directive principles specified in Par tIV of the Constitution and not merely in Article 39 (b) or (c).

    However, this extension was declared as unconstitutional by the Supreme Court in the Minerva Mills case (1980).


24)   Which of the following is/are true?

1) Declaration of martial law results in the suspension of the writ of habeas corpus.
2) Article 33 empowers the Parliament to restrict the fundamental rights of the members of armed forces.


a. Only 1
b. Only 2
c. Both 1 and 2
d. Neither 1 nor 2
Answer  Explanation 

ANSWER: Only 2

Explanation:

  • Recently there were videos posted by soldiers on social media sites complaining about their conditions.

  • Article 33 empowers the Parliament to restrict or abrogate the fundamental rights of the members of armed forces, para-military forces, police forces, intelligence agencies and analogous forces.

  • The objective is to ensure the proper discharge of their duties and the maintenance of discipline among them.

  • The power to make laws under Article 33 is conferred only on Parliament.

  • Any such law made by Parliament cannot be challenged in any court on the ground of contravention of any of the fundamental rights.

  • Various Acts enacted thus, impose restrictions on their freedom of speech, right to form associations, right to be members of trade unions or political associations, right to communicate with the press, right to attend public meetings or demonstrations etc.

  • The expression ‘members of the armed forces’ also covers such employees of the armed forces as barbers, carpenters, mechanics, cooks, chowkidars, bootmakers, tailors who are non-combatants.

  • A parliamentary law enacted under Article 33 can also exclude the court martials (tribunals established under the military law) from the writ jurisdiction of the Supreme Court and the high courts, so far as the enforcement of Fundamental Rights is concerned.

  • Article 34 provides for the restrictions on fundamental rights while martial law is in force in any area within the territory of India.

  • It empowers the Parliament to indemnify any government servant or any other person for any act done by him in connection with the maintenance or restoration of order in any area where martial law was in force.

  • The Parliament can also validate any sentence passed,punishment inflicted or other act done under martial law in such area.

  • The Act of Indemnity made by the Parliament cannot be challenged in any court on the ground of contravention of any of the fundamental rights.

  • The concept of martial law has been borrowed in India from the English common law.


  • The expression ‘martial law’ has not been defined anywhere in the Constitution.

  • It means ‘military rule’.

  • Here civil administration is run by the military authorities according to their own rules and regulations framed outside the ordinary law.

  • It is different from the military law that is applicable to the armed forces.

  • There is also no specific or express provision in the Constitution that authorizes the executive to declare martial law.

  • But, it is implicit in Article 34 under which martial law can be declared in any area within the territory of India.

  • The martial law is imposed under the extraordinary circumstances like war, invasion, insurrection, rebellion, riot or any violent resistance to law.

  • Its justification is to repel force by force for maintaining or restoring order in the society.

  • During the operation of martial law, the military authorities are vested with abnormal powers to take all necessary steps.

  • They impose restrictions and regulations on the rights of the civilians, can punish the civilians and even condemn them to death.

  • The Supreme Court held that the declaration of martial law does not ipso facto result in the suspension of the writ of habeas corpus.


25)   Which of the following are correctly matched?

1) Certiorari - to forbid
2) Mandamaus - we command
3) Quo-Warranto - by what authority


a. 2, 3
b. 1, 3
c. 1, 2
d. All of the above
Answer  Explanation 

ANSWER: 2, 3

Explanation:

  • Habeas Corpus - It’s Latin, which literally means ‘to have the body of’.
  • It is an order issued by the court to a person who has detained another person, to produce the body of the latter before it.
  • The court then examines the cause and legality of detention.
  • It would set the detained person free, if the detention is found to be illegal.
  • Thus, this writ is a bulwark of individual liberty against arbitrary detention.
  • The writ of habeas corpus can be issued against both public authorities as well as private individuals.

  • The writ is not issued where the -

    1. detention is lawful;
    2. the proceeding is for contempt of a legislature or a court;
    3. detention is by a competent court; and
    4. detention is outside the jurisdiction of the court.

  • Mandamus -It means ‘we command’.
  • It is a command issued by the court to a public official asking him to perform his official duties that he has failed or refused to perform.
  • It can also be issued against any public body, a corporation, an inferior court, a tribunal or government for the same purpose.

  • The writ of mandamus cannot be issued -

    1. against a private individual or body;
    2. to enforce departmental instruction that does not possess statutory force;
    3. when the duty is discretionary and not mandatory;
    4. to enforce a contractual obligation;
    5. against the president of India or the state governors; and
    6. against the chief justice of a high court acting in judicial capacity.

  • Prohibition -It means ‘to forbid’.
  • It is issued by a higher court to a lower court or tribunal to prevent the latter from exceeding its jurisdiction or usurping a jurisdiction.
  • Mandamus directs activity, but prohibition directs inactivity.
  • The writ of prohibition can be issued only against judicial and quasi-judicial authorities.
  • It is not available against administrative authorities, legislative bodies, and private individuals or bodies.

  • Certiorari - It means ‘to be certified’ or ‘to be informed’.
  • It is issued by a higher court to a lower court or tribunal either to transfer a case pending with the latter to itself or to squash the orderof the latter in a case.
  • It is issued on the grounds of excess of jurisdiction or lack of jurisdiction or error of law.
  • Prohibition is only preventive, but certiorari is both preventive as well as curative.
  • Till recently, the writ of certiorari could be issued only against judicial and quasi-judicial authorities.
  • In 1991, the Supreme Court ruled that the certiorari can be issued even against administrative authorities affecting rights of individuals.
  • Like prohibition, certiorari is also not available against legislative bodies and private individuals or bodies.

  • Quo-Warranto - It means ‘by what authority or warrant’.
  • It is issued by the court to enquire into the legality of claim of a person to a public office.
  • It prevents illegal usurpation of public office by a person.
  • The writ can be issued only in case of a substantive public office of a permanent character created by a statute or by the Constitution.
  • It cannot be issued in cases of ministerial office or private office.
  • Unlike the other four writs, this can be sought by any interested person and not necessarily by the aggrieved person.


26)   Which of the following is/are true?

1) Writs have been borrowed from English law.
2) Territorial jurisdiction of high court to issue writs is lower than that of Supreme court
3) Writ jurisdiction of high court of lower than that of Supreme court
4) Supreme Court cannot refuse to exercise its writ jurisdiction


a. 1, 2 and 4
b. 1, 2 and 3
c. 3 and 4
d. All of the above
Answer  Explanation 

ANSWER: 1, 2 and 4

Explanation:

  • The Supreme Court (under Article 32) and the high courts (under Article 226) can issue the writs of habeas corpus, mandamus, prohibition, certiorari and quo-warranto.
  • The Parliament (under Article 32) can empower any other court to issue these writs.
  • Before 1950, only the High Courts of Calcutta, Bombay and Madras had the power to issue the writs.
  • Article 226 now empowers all the high courts to issue the writs.
  • These writs are borrowed from English law where they are known as ‘prerogative writs’.
  • They are so called in England as they were issued in the exercise of the prerogative of the King who was, andis still, described as the ‘fountain of justice’.

  • The writ jurisdiction of the Supreme Court differs from that of a high court in three respects :

    1. The Supreme Court can issue writs only for the enforcement of fundamental rights whereas a high court can issue writs not only for the enforcement of Fundamental Rights but also for any other purpose. Thus, the writ jurisdiction of the Supreme Court, in this respect, is narrower than that of high court.

    2. The Supreme Court can issue writs against a person or government throughout the territory of India whereas a high court can issue writs against a person residing or against a government or authority located within its territorial jurisdiction only or outside its territorial jurisdiction only if the cause of action arises within its territorial jurisdiction. Thus, the territorial jurisdiction of the Supreme Court for issuing writs is wider than that of a high court.

    3. A remedy under Article 32 is in itself a Fundamental Right and hence, the Supreme Court may not refuse to exercise its writ jurisdiction. On the other hand, a remedy under Article 226 is discretionary and hence, a high court may refuse to exercise its writ jurisdiction. Article 32does not merely confer power on the Supreme Court as Article 226 does on a high court to issue writs. The Supreme Court is constituted as a defender and guarantor of the fundamental rights.


27)   Freedom of speech and expression includes

1) Right to strike
2) Freedom of commercial advertisements
3) Freedom of silence


a. 2, 3
b. 1, 2
c. 1, 3
d. All of the above
Answer  Explanation 

ANSWER: 2, 3

Explanation:

  • Article 19 guarantees to all citizens six rights.

  • These are :
    1. Right to freedom of speech and expression.
    2. Right to assemble peaceably and without arms.
    3. Right to form associations or unions or co-operative societies.
    4. Right to move freely throughout the territory of India.
    5. Right to reside and settle in any part of the territory of India.
    6. Right to practice any profession or to carry on any occupation, trade or business.

  • Originally, Article 19 contained seven rights.

  • But, the right to acquire, hold and dispose of property was deleted by the 44th Amendment Act of 1978.

  • These six rights are protected against only state action and not private individuals.

  • Moreover, these rights are available only to the citizens and to shareholders of a company but not to foreigners or legal persons like companies or corporations, etc.

  • The State can impose ‘reasonable’ restrictions on the enjoyment of these six rights only on the grounds mentioned in the Article 19 itself and not on any other grounds.

  • Freedom of Speech and Expression - It implies that every citizen has the right to express his views,opinions, belief and convictions freely by word of mouth, writing, printing, picturing or in any other manner.

  • The Supreme Court held that the freedom of speech and expression includes the following :

    1. Right to propagate one’s views as well as views of others.
    2. Freedom of the press.
    3. Freedom of commercial advertisements.
    4. Right against tapping of telephonic conversation.
    5. Right to telecast, that is, government has no monopoly on electronic media.
    6. Right against bundh called by a political party or organization.
    7. Right to know about government activities.
    8. Freedom of silence.
    9. Right against imposition of pre-censorship on a newspaper.
    10. Right to demonstration or picketing but not right to strike.

  • The State can impose reasonable restrictions on the exercise of the freedom of speech and expression on the grounds of sovereignty and integrity of India, security of the state, friendly relations with foreign states, public order, decency or morality, contempt of court, defamation, and incitement to an offence.


28)   Which of the following is/are true?

1) Freedom of assembly includes right to strike.
2) Right to obtain recognition of association is not a fundamental right.
3) Article 19 protects only right to move inside the country.


a. 2, 3
b. 1, 2
c. 1, 3
d. All of the above
Answer  Explanation 

ANSWER: 2, 3

Explanation:

  • Freedom of Assembly - Every citizen has the right to assemble peaceably and without arms.

  • It includes the right to hold public meetings, demonstrations and take out processions.

  • This freedom can be exercised only on public land and the assembly must be peaceful and unarmed.

  • This provision does not protect violent, disorderly, riotous assemblies, or one that causes breach of public peace or one that involves arms.

  • This right does not include the right to strike.

  • The State can impose reasonable restrictions on the exercise of right of assembly on two grounds,namely, sovereignty and integrity of India and public order including the maintenance of traffic in the area concerned.

  • Under Section 144 of Criminal Procedure Code (1973), a magistrate can restrain an assembly,meeting or procession if there is a risk of obstruction, annoyance or danger to human life, health or safety or a disturbance of the public tranquility or a riot or any affray.

  • Under Section 141 of the Indian Penal Code, as assembly of five or more persons becomes unlawful if the object is :

  • 1. to resist the execution of any law or legal process;
    2. to forcibly occupy the property of some person;
    3. to commit any mischief or criminal trespass;
    4. to force some person todo an illegal act; and
    5. to threaten the government or its officials on exercising lawful powers.

  • Freedom of Association - All citizens have the right to form associations or unions or co-operative societies. It includes the right to form political parties, companies, partnership firms, societies,clubs, organizations, trade unions or any body of persons.

  • It not only includes the right to start an association or union but also to continue with the association or union as such.

  • Further, it covers the negative right of not to form or join an association or union.

  • Reasonable restrictions can be imposed on the exercise of this right by the State on the grounds of sovereignty and integrity of India, public order and morality.

  • Subject to these restrictions, the citizens have complete liberty to form associations or unions for pursuing lawful objectives and purposes.

  • However, the right to obtain recognition of the association is not a fundamental right.

  • The Supreme Court held that the trade unions have no guaranteed right to effective bargaining or right to strike or right to declare a lock-out.

  • The right to strike can be controlled by an appropriate industrial law.

  • Freedom of Movement - This freedom entitles every citizen to move freely throughout the territory of the country.

  • He can move freely from one state to another or from one place to another within a state.

  • This right underline the idea that India is one unit so far as the citizens are concerned.

  • Thus, the purpose is to promote national feeling and not parochialism.

  • The grounds of imposing reasonable restrictions on this freedom are two, namely, the interests of general public and the protection of interests of any scheduled tribe. The entry of outsiders in tribal areas is restricted to protect the distinctive culture, language, customs and manners of scheduled tribes and to safeguard their traditional vocation and properties against exploitation.

  • The Supreme Court held that the freedom of movement of prostitutes can be restricted on the ground of public health and in the interest of public morals.

  • The Bombay High Court validated the restrictions on the movement of persons affected by AIDS.

  • The freedom of movement has two dimensions, viz, internal (right to move inside the country) and external (right to move out of the country and right to come back to the country).

  • Article 19 protects only the first dimension.

  • The second dimension is dealt by Article 21 (right to life and personal liberty).


29)   Which of the following is/are true?

1) The right of outsiders to reside and settle in tribal areas is restricted.
2) No objection can be made when the State carries on a trade, business, etc. as a monopoly.


a. Only 1
b. Only 2
c. Both 1 and 2
d. Neither 1 nor 2
Answer  Explanation 

ANSWER: Both 1 and 2

Explanation:

  • Freedom of Residence - Every citizen has the right to reside and settle in any part of the territory of the country.

  • This right has two parts :

    1. the right to reside in any part of the country, which means to stay at any place temporarily, and
    2. the right to settle in any part of the country, which means to set up a home or domicile at any place permanently.

  • This right is intended to remove internal barriers within the country or between any of its parts.

  • This promotes nationalism and avoids narrow mindedness.

  • The State can impose reasonable restrictions on the exercise of this right on two grounds, namely, the interest of general public and the protection of interests of any scheduled tribes.

  • The right of outsiders to reside and settle in tribal areas is restricted to protect the distinctive culture, language, customs and manners of scheduled tribes and to safeguard their traditional vocation and properties against exploitation.

  • In many parts of the country, the tribals have been permitted to regulate their property rights in accordance with their customary rules and laws.

  • The Supreme Court held that certain areas can be banned for certain kinds of persons like prostitutes and habitual offenders.

  • From the above, it is clear that the right to residence and the right to movement are overlapping to some extent.

  • Both are complementary to each other.

  • Freedom of Profession, etc. - All citizens are given the right to practice any profession or to carry on any occupation, trade or business.

  • This right is very wide as it covers all the means of earning one’s livelihood.

  • The State can impose reasonable restrictions on the exercise of this right in the interest of the general public.

  • Further, the State is empowered to -

    1. prescribe professional or technical qualifications necessary for practicing any profession or carrying on any occupation, trade or business; and
    2. carry on by itself any trade, business, industry or service whether to the exclusion (complete or partial) of citizens or otherwise.

  • Thus, no objection can be made when the State carries on a trade, business, industry or service either as a monopoly (complete or partial) to the exclusion of citizens (all or some only) or in competition with any citizen.

  • The State is not required to justify its monopoly.

  • This right does not include the right to carry on a profession or business or trade or occupation that is immoral (trafficking in women or children) or dangerous (harmful drugs or explosives, etc,).

  • The State can absolutely prohibit these or regulate them through licensing.


30)   Which of the following is/are true?

1) As per article 15, discrimination on grounds other than religion, race, caste, sex or place of birth is allowed.
2) Group ‘B’/Class II Officers of the All India, Central and State Services do not come under creamy layer of OBC.


a. Only 1
b. Only 2
c. Both 1 and 2
d. Neither 1 nor 2
Answer  Explanation 

ANSWER: Only 1

Explanation:

  • Article 15 provides that the State shall not discriminate against any citizen on grounds only of religion, race, caste, sex or place of birth.

  • The two crucial words in this provision are ‘discrimination’ and ‘only’.

  • The word ‘discrimination’ means ‘to make an adverse distinction with regard to’ or ‘to distinguish unfavorably from others’.

  • The use of the word ‘only’ connotes that discrimination on other grounds is not prohibited.

  • The second provision of Article 15 says that no citizen shall be subjected to any disability, liability,restriction or condition on grounds only of religion, race, caste, sex, or place of birth with regard to :

    1. access to shops, public restaurants, hotels and places of public entertainment; or
    2. the use of wells, tanks, bathing ghats, road and places of public resort maintained wholly or partly by State funds or dedicated to the use of general public.

  • This provision prohibits discrimination both by the State and private individuals, while the former provision prohibits discrimination only by the State.

  • There are three exceptions to this general rule of non-discrimination :

    1. The state is permitted to make any special provision for women and children. For example,reservation of seats for women in local bodies or provision of free education for children.

    2. The state is permitted to make any special provision for the advancement of any socially and educationally backward classes of citizens or for the scheduled castes and scheduled tribes. For example, reservation of seats or fee concessions in public educational institutions.

    3. The state is empowered to make any special provision for the advancement of any socially and educationally backward classes of citizens or for the scheduled castes or the scheduled tribes regarding their admission to educational institutions including private educational institutions, whether aided or unaided by the state, except the minority educational institutions.

  • The last provision was added by the 93rdAmendment Act of 2005.

  • In order to give effect to this provision, the Centre enacted the Central Educational Institutions (Reservation in Admission) Act,2006, providing a quota of 27% for candidates belonging to the Other Backward Classes (OBCs) in all central higher educational institutions including the IITs and IIMs.

  • In April 2008, the Supreme Court upheld the validity of both,the Amendment Act and the OBC Quota Act.

  • But, the Court directed the central government to exclude the ‘creamy layer’ (advanced sections) among the OBCs while implementing the law.

  • Creamy Layer - The children of the following different categories of people belong to ‘creamy layer’ among OBCs and thus will not get the quota benefit -

  • a. Persons holding constitutional posts like President, Vice-President, Judges of SC and Hcs, Chairman and Members of UPSC and SPSCs, CEC, CAG and so on.

    b. Group ‘A’ / Class I and Group ‘B’ / Class II Officers of the All India, Central and State Services; and Employees holding equivalent posts in PSUs, Banks, Insurance Organisations, Universities etc., and also in private employment.

    c. Persons who are in the rank of colonel and above in the Army and equivalent posts in the Navy,the Air Force and the Paramilitary Forces.

    d. Professionals like doctors, lawyers, engineers, artists, authors, consultants and so on.

    e. Persons engaged in trade, business and industry.

    f. People holding agricultural land above a certain limit and vacant land or buildings in urban areas.

    g. Persons having gross annual income of more than Rs.4.5 lakhs or possessing wealth above the exemption limit.

  • In 1993, when the “creamy layer” ceiling was introduced, it was Rs. 1 lakh. It was revised to Rs. 2.5 lakh in 2004 and Rs. 4.5 lakh in 2008.

  • Presently (2013), the proposal to raise creamy layer ceiling to Rs.6 lakh a year is under consideration of the government.


31)   Who was to identify creamy layer amongst OBCs?

a. Kaka Kalelkar Committee
b. National Commission for Backward Classes
c. Ram Nandan Committee
d. Mandal Commission
Answer  Explanation 

ANSWER: Ram Nandan Committee

Explanation:

  • Mandal Commission - In 1979, the Morarji Desai Government appointed the Second Backward Classes Commission under the chairmanship of B P Mandal, a Member of Parliament.

  • It was as per terms of Article 340 of the Constitution to investigate the conditions of the socially and educationally backward classes and suggest measures for their advancement.

  • The commission submitted its report in 1980 and identified as many as 3743 castes as socially and educationally backward classes.

  • They constitute nearly 52% component of the population, excluding the scheduled castes (SCs) and the scheduled tribes (STs).

  • The first Backward Classes Commission was appointed in 1953 under the chairmanship of Kaka Kalelkar. It submitted its report in 1955.

  • In 1963, the Supreme Court ruled that more than 50% reservation of jobs in a single year would be unconstitutional.

  • The Mandal commission recommended for reservation of 27% government jobs forthe Other Backward Classes (OBCs) so that the total reservation for all ((SCs, STs and OBCs)amounts to 50%.

  • It was after ten years in 1990 that the V P Singh Government declared reservation of 27% government jobs for the OBCs.

    Again in 1991, the Narasimha Rao Government introduced two changes :

    1. preference to the poorer sections among the OBCs in the 27% quota, i.e., adoption of the economic criteria in granting reservation; and

    2. reservation of another 10% of jobs for poorer(economically backward) sections of higher castes who are not covered by any existing schemes of reservation.

  • In the famous Mandal case (1992), the scope and extent of Article 16(4), which provides for reservation of jobs in favour of backward classes, has been examined thoroughly by the Supreme Court.

  • Though the Court has rejected the additional reservation of 10% for poorer sections of higher castes, it upheld the constitutional validity of 27% reservation for the OBCs with certain conditions,viz,

  • 1. The advanced sections among the OBCs (the creamy layer) should be excluded from the list of beneficiaries of reservation.

    2. No reservation in promotions; reservation should be confined to initial appointments only. Any existing reservation in promotions can continue for five years only (i.e., upto 1997).

    3. The total reserved quota should not exceed 50% except in some extraordinary situations. This rule should be applied every year.

    4. The ‘carry forward rule’ in case of unfilled (backlog) vacancies is valid. But it should not violate 50% rule.

    5. A permanent statutory body should be established to examine complaints of over-inclusion and under-inclusion in the list of OBCs.

    With regard to the above rulings of the Supreme Court, the government has taken the following action :

    1. Ram Nandan Committee was appointed to identify the creamy layer among the OBCs. It submitted its report in 1993, which was accepted.

    2. National Commission for Backward Classes was established in 1993 by an act of Parliament. It considers inclusions in and exclusions from the lists of castes notified as backward for the purpose of job reservation.

    3. In order to nullify the ruling with regard to reservation in promotions, the 77th Amendment Act was enacted in 1995. It added a new provision in Article 16 that empowers the State to provide for reservation in promotions of any services under the State in favor of the SCs and STs that are not adequately represented in the state services. Again, the 85th Amendment Act of 2001 provides for ‘consequential seniority’ in the case of promotion by virtue of rule of reservation for the government servants belonging to the SCs and STs with retrospective effect from June 1995.

    4. The ruling with regard to backlog vacancies was nullified by the 81st Amendment Act of 2000. It added another new provision in Article 16 that empowers the State to consider the unfilled reserved vacancies of a year as a separate class of vacancies to be filled up in any succeeding year or years. Such class of vacancies are not to be combined with the vacancies of the year in which they are being filled up to determine the ceiling of 50% reservation on total number of vacancies of that year. In brief, it ends the 50% ceiling on reservation in backlog vacancies.

    5. The 76th Amendment Act of 1994 has placed the Tamil Nadu Reservations Act of 1994 in the Ninth Schedule to protect it from judicial review as it provided for 69 per cent of reservation, far exceeding the 50 per cent ceiling.


32)   The scope of Article 29 includes -

1) Religious minorities
2) Linguistic minorities
3) Majority


a. Only 1 and 2
b. Only 1 and 3
c. Only 2 and 3
d. All of the above
Answer  Explanation 

ANSWER: All of the above

Explanation:

  • Article 29 provides that any section of the citizens residing in any part of India having a distinct language, script or culture of its own, shall have the right to conserve the same.

  • No citizen shall be denied admission into any educational institution maintained by the State or receiving aid out of State funds on grounds only of religion, race, caste, or language.

  • The first provision protects the right of a group.

  • The second provision guarantees the right of a citizen as an individual irrespective of the community to which he belongs.

  • Article 29 grants protection to both religious minorities as well as linguistic minorities.

  • However, the Supreme Court held that the scope of this article is not necessarily restricted to minorities only, as it is commonly assumed to be.

  • This is because of the use of words ‘section of citizens’ in the Article that include minorities as well as majority.

  • The Supreme Court also held that the right to conserve the language includes the right to agitate for the protection of the language.

  • Hence, the political speeches or promises made for the conservation of the language of a section of the citizens does not amount to corrupt practice under the Representation of the People Act, 1951.


33)   Which of the following are Fundamental Duties?

1) Safeguard public property
2) Have compassion for living creatures
3) Provide opportunities for education to his child or ward between the age of 6 and 14 years


a. Only 1 and 2
b. Only 1 and 3
c. Only 2, 3 and 4
d. All of the above
Answer  Explanation 

ANSWER: All of the above

Explanation:
According to Article 51 A, it shall be the duty of every citizen of India -

1. to abide by the Constitution and respect its ideals and institutions, the National Flag and the National Anthem;

2. to cherish and follow the noble ideals that inspired the national struggle for freedom;

3. to uphold and protect the sovereignty, unity and integrity of India;
to defend the country and render national service when called upon to do so;

4. to promote harmony and the spirit of common brotherhood amongst all the people of India transcending religious, linguistic and regional or sectional diversities and to renounce practices derogatory to the dignity of women;

5. to value and preserve the rich heritage of the country’s composite culture;

6. to protect and improve the natural environment including forests, lakes, rivers and wild life and to have compassion for living creatures;

7. to develop scientific temper, humanism and the spirit of inquiry and reform;

8. to safeguard public property and to abjure violence;

9. to strive towards excellence in all spheres of individual and collective activity so that the nation constantly rises to higher levels of endeavor and achievement; and

10. to provide opportunities for education to his child or ward between the age of six and fourteen years. (added by the 86th Constitutional Amendment Act, 2002).

  • Some of them are moral duties while others are civic duties.
  • They refer to values which have been a part of the Indian tradition, culture, religions and practices.
  • They essentially contain just a codified version of tasks integral to the Indian way of life.
  • Fundamental Duties are confined to citizens only and do not extend to foreigners.
  • Fundamental duties are also non-justiciable.
  • However, the Parliament is free to enforce them by suitable legislation.


34)   Which of the following can be considered as legal provisions for the implementation of some of the Fundamental Duties?

1) The Forest (Conservation) Act of 1980
2)Prevention of Insults to National Honour Act
3)Wildlife (Protection) Act of 1972
4)Protection of Civil Rights Act (1955)


a. 1, 2, 3
b. 3, 4
c. 2, 3 and 4
d. All of the above
Answer  Explanation 

ANSWER: All of the above

Explanation:

  • The Verma Committee on Fundamental Duties of the Citizens (1999) found legal provisions for the implementation of some of the Fundamental Duties.

  • Some of them are -

    1. The Prevention of Insults to National Honour Act (1971) - prevents disrespect to the Constitution of India, the National Flag and the National Anthem.

    2. Various criminal laws in force - for punishments for encouraging enmity between different sections of people on grounds of language, race, place of birth, religion, etc.

    3. The Protection of Civil Rights Act (1955) - for punishments for offences related to caste and religion.

    4. The Indian Penal Code (IPC) mentions the assertions prejudicial to national integration as punishable offences.

    5. The Unlawful Activities (Prevention) Act of 1967 - for the declaration of a communal organization as an unlawful association.

    6. The Representation of People Act (1951) - for the disqualification of members of the Parliament or a state legislature for indulging in corrupt practice, that is, soliciting votes on the ground of religion or promoting enmity between different sections of people.

    7. The Wildlife (Protection) Act of 1972 - prohibits trade in rare and endangered species.

    8. The Forest (Conservation) Act of 1980 - checks indiscriminate deforestation and diversion of forest land for non-forest purposes.


35)   Which of the following is/are true regarding Fundamental Duties?

1) Their inclusion in the Constitution is unnecessary as per some critics.
2) The Parliament can provide for appropriate penalty or punishment for failure to fulfill any of the Fundamental Duties.


a. Only 1
b. Only 2
c. Both 1 and 2
d. Neither 1 nor 2
Answer  Explanation 

ANSWER: Both 1 and 2

Explanation:

    Significance of Fundamental Duties -

    1. Remind the citizens to be conscious of duties while enjoying their rights.

    2. Serve as a warning against the anti-national and antisocial activities.

    3. Serve as a source of inspiration for the citizens and promote a sense of discipline and commitment.

    4. They create a feeling that the citizens are active participants in the realization of national goals.

    5. They help the courts in examining and determining the constitutional validity of a law.

    6. In 1992, the Supreme Court ruled that in determining the constitutionality of any law, if a court finds that the law in question seeks to give effect to a fundamental duty, it may consider such law to be ‘reasonable’ in relation to Article 14 (equality before law) or Article 19 (six freedoms) and thus save such law from unconstitutionality.

    7. They are enforceable by law. The Parliament can provide for appropriate penalty or punishment for failure to fulfill any of them.

    8. Indira Gandhi, the then Prime Minister, justified the inclusion of fundamental duties in the Constitution by saying that their inclusion helps to strengthen democracy.

    9. The opposition in the Parliament strongly opposed the inclusion of fundamental duties by the Congress government.

    10. The new Janata Government headed by Morarji Desai in the post-emergency period did not abolish the Fundamental Duties.

    11. This proves the necessity of including the Fundamental Duties in the Constitution.

    12. This is proven by adding one more Fundamental Duty in 2002 by the 86thAmendment Act.

    Criticism of Fundamental Duties -

    1. The list of duties is not exhaustive. It does not cover other important duties like casting vote, paying taxes, family planning, etc. (duty to pay taxes was recommended by the Swaran Singh Committee)

    2. Some of the duties are vague, ambiguous and difficult to understand by the common man.

    3. Since they are non-justiciable they have been described by the critics as a code of moral guidelines. (Swaran Singh Committee had suggested for penalty or punishment for the non-performance of Fundamental Duties)

    4. Their inclusion in the Constitution is unnecessary as per critics as these included duties would be performed by the people even if they were not in the Constitution.

    5. Adding fundamental duties as an appendage to Part IV has reduced their significance.

    6. They should be after Part III, on par with Fundamental Rights.


36)   Which of the following is/are true?

1) Japanese constitution is the only democratic constitution in world which contains a list of duties of citizens.
2) Swaran Singh Committee suggested duty to pay taxes as a fundamental duty.


a. Only 1
b. Only 2
c. Both 1 and 2
d. Neither 1 nor 2
Answer  Explanation 

ANSWER: Both 1 and 2

Explanation:

  • Duties of the State are incorporated in the Constitution in the form of Directive Principles of State Polity.
  • In 1976, the fundamental duties of citizens were added in the Constitution.
  • The Fundamental Duties in the Indian Constitution are inspired by the Constitution of erstwhile USSR.
  • However, none of the Constitutions of major democratic countries specifically contain a list of duties of citizens.
  • Japanese Constitution is the only democratic Constitution in world which contains a list of duties of citizens.
  • The socialist countries, gave equal importance to the fundamental rights and duties of their citizens.

  • Swaran Singh Committee Recommendations -

    1. In 1976, the Congress Party set up the Sardar Swaran Singh Committee to make recommendations about fundamental duties.
    2. Its necessity was felt during the operation of the internal emergency (1975–1977).
    3. The Congress Government at Centre accepted its recommendations and enacted the 42nd Constitutional Amendment Act in 1976.
    4. This amendment added a new part, Part IVA to the Constitution.
    5. This new part consists of only one Article, that is, Article 51A.
    6. Swaran Singh Committee suggested the incorporation of 8 Fundamental Duties in the Constitution but the 42nd Constitutional Amendment Act (1976) included 10 Fundamental Duties.
    7. Certain recommendations of the Committee were not accepted.

    These include :

    (i) The Parliament may provide for the imposition of penalty or punishment for not observing any of the duties.
    (ii) No law imposing such punishment can be questioned in any court on the ground of infringement of any of Fundamental Rights or on the ground of repugnancy to any other provision of the Constitution.
    (iii) Duty to pay taxes.